Hacker News new | past | comments | ask | show | jobs | submit login
Continuing frequency deviation in the Continental European Power System (entsoe.eu)
478 points by Aissen on March 7, 2018 | hide | past | favorite | 322 comments



The short version of the story is that grid electricity in Europe is produced by rotating the turbines 50 times a second and that's where the 50Hz AC electricity comes from.

Apparently to match the demand at given instance electricity plants rotate their turbines a bit faster or a bit slower instead of switching a complete plant on and off[edit: this is not exactly right, see the reply below], which results in slight deviations from the 50Hz standard but it is fine as long as it stays between the limits.

At the end of the day, as the demand increases and decreases, the average frequency would be 50Hz and engineers took advantage of that fact to create clocks that may not be accurate to the second but accurate on average. How do they do that? They count the change in the electricity and assume that 50 changes are exactly 1 second.

Unfortunately, due to political issues in the Balkans, the grid was undersupplied or oversupplied for a prolonged period and this created a deviation from the average of 50Hz and the clocks that depend on this average to be 50Hz also lost accuracy that currently amounts to 6 minutes.


> Apparently to match the demand at given instance electricity plants rotate their turbines a bit faster or a bit slower instead of switching a complete plant on and off, which results in slight deviations from the 50Hz standard but it is fine as long as it stays between the limits.

This is slightly imprecise. It actually works the other way round: the demand is "communicated" through the network by slight changes in the frequency.

This can be described best for the example of steam-based generators, as they are typical for the majority of coal/gas/oil/nuclear power plants. When demand for electricity grows, for example because a new factory is powered up, the turbines in the power plants are experiencing larger forces attempting to slow them down, starting with the power plant closest to the new factory.

The speed (and thus the frequency) is constantly measured, and if it drops, the steam valve is opened just a tiny bit more to counteract the drop. If this is not possible because the power plant is already at maximum output, either other power plants have to take the load (and the electricity is then routed through the network to the factory) or additional power plants may have to be added to the network.

The important thing is: the frequency change is a means of communication, it originates from demand changes, and because the entire network is kept in sync, it works to communicate demand changes across the entire network. Power plants providing energy to a sub-part of the entire network always attempt to run their turbines with the exact same speed of 50 rotations per second, which should ideally always conform to the frequency in all other sub-networks that a specific sub-network is attached to.

If just one of these sub-networks does not perform its duty of re-establishing 50 Hertz by powering up their own plant output, the rest of the network must either channel enough power into this sub-network to allow its local generators to re-establish 50 rotations per second by taking some of the load off of them or - if that is not possible or decided against - must also deliberately drop their frequency (and thus the rotational speed of their generators) to match the sub-network that is deviating.

The only alternative to this would be to drop off the deviating sub-network entirely, but that is usually only done in extreme cases of deviation.


I have a question about the power grid I've never gotten a satisfactory answer to. I hope someone here can help.

If the USA's power grid is about 4,000,000 meters wide, and the electric signal propagates at roughly the speed of light (3 x 10^8 m/s) (https://physics.stackexchange.com/questions/358894/speed-of-...), then it will take roughly .013 seconds for the electrical signal to reach from one station to another. This is a lot out of phase! (the period of 60 Hz is .016 sec) How is this managed?

If two generating stations on opposite ends of the country are both contributing to the electrical signal, how is this time lag accounted for? I have two guesses:

1. The power network is "mapped" such that there is one central generating station and all other generating stations are time lagged based on their graph distance from this generating station

2. All stations try to generate simultaneously, with their output signals interfering with eachother to a certain degree. This might not matter much because their local loads dampen the strength of their signal.

Either way seems to introduce inefficiencies.


Synchronisation is purely local. They are, from the point of view of a sky observer, out of phase by the amount you say, but that doesn't matter. Each generator need only sync up at the time of connection.

Imagine you are on half a tandem bicycle. You are in a room with the chain of the tandem vanishing beyond the wall. You can't see, but clearly someone else is pedalling the thing as the pedals are rotating. Getting on it is tricky, as you have to rotate your feet at the right speed, but once you're on you can sit there and let it carry them round - or you can start applying pressure through your feet to do work and accelerate the chain system.

At this point you discover the thing on the other end of the chain is not a person but a 60Hz synchronous motor-generator. Congratulations, you are in sync with the grid and (when pushing on the pedals) contributing power.

(A corollary of this is that if your grid connection is down or the grid is split into two pieces, you can't start up again until you get a grid input to know you're in phase. See "black start" for details on this.)


Okay, but what if you are at point A, and connected to points B and C, which are 1000 km away from you and also 1000 km away from each other, and connected to each other. Now if you would synchronize with B and C, then seen from the sky B and C would be in sync with each other because they're at the same distance from you. But how would B and C then transfer power over the 1000 km link between each other without phase problems?


Short version: it doesn't.

It's like a very long pipe with standing water in it. AC power is like attaching a pump column at one point and adding/removing pressure at just that point. This creates a /wave/ of energy that propagates, but the actual water (electrons) do not. It's more accurate to say they vibrate.

Continuing the metaphor, power plants increase the amplitude of this sloshing wave, and must be synchronized to the phase of the wave /where they connect/ (in order to be additive instead of subtractive). Consumers of the power are always subtractive, '(active/passive) power factor correction' in switching power supplies is about attempting to keep the consumption timed for efficiency. AC Motors are continuous users and mostly just draw all the time.


Remember that power stations do not generate the same single phase signal that you tap into in your house. They generate three phase AC current. They also don't link up directly: there are transmission substations in between.


This is what I wondered, is network topology carefully arranged to avoid triangular connections, or 'loops' where a generator could interfere (indirectly) with itself?


Today the physical topology is usually somewhat random mesh, but not all interconnection lines are active at the same time.

In early days of electrification the physical topology usually was spanning tree and in fact this was the motivation for research into minimal spanning tree algorithms at the time.


Nope! They're connected in triangles all the time. I was stuck on this phase synchronization for a while, but really it's just the local phase lead or lag that is required.

Think of the phase as a distributed signal that indicates the current state of the local system taking into account all connected generators. It's weird but true!


Can you elaborate? It is still unclear to me.


I may be explaining it wrong, but here is a reference, particularly 22.8:

https://www.scribd.com/doc/118169687/Network-Protection-Auto...

This makes it sound more complex than what I learned when I last tried to figure it out =)

They seem to be describing that where two grids meet they go through a synchronizer that disconnects one or the other if they aren't in sync, so implies manually disconnections throughout the network as needed locally.


That doesn’t answer his question.


This explanation is necessarily wrong. There exist multiple paths of different phase lengths between two nodes (eg in a triangle configuration), unlike your bicycle example. In particular, the L1 and L2 distance metrics are equivalent on a line but not on a 2D plane or sphere.


A fun concrete aspect of this is the "synchroscope" used when bringing generators online: https://en.wikipedia.org/wiki/Synchroscope


My confusion always comes when I try to think of it as a propagating wave.

If instead of a tandem bicycle, it is a very long rope that someone is moving up and down, then when I try to join some distance away there will seem to be a wave passing me. Even if I join in at the correct phase, I will also be generating a wave traveling back to the first 'generator' and interfering with the existing wave along the way. With the right distance and relative strength then we will produce a standing wave between us. Is there an apparent 'direction' to the wave of electric potential in a large grid?

Then there is the issue of triangular arrangements the sibling comment raised, when the radial distances and relative phases don't align.

Maybe my issue is always trying to think through physical analogues I can 'see'.


Note that the wavelength of 50Hz at 'c' is 6000km. There's not a lot of space to get very much out of phase and it will attenuate along the way.

The nearest I can think to "direction" is "reactive power", or the phase angle between voltage and current at any particular point.

There very definitely are loops and multiple paths to a particular point, it's not called a "grid" for nothing. This little map of the UK grid is interesting: https://www.nationalgrid.com/uk/about-grid/our-networks-and-...

Edit: Aha, further reading - systems for altering the phase angle: https://en.wikipedia.org/wiki/Quadrature_booster

And https://www.researchgate.net/publication/27341916_Influence_...


Along with quadrature boosters, there are (used to be?) these things, synchronous condensers: https://en.wikipedia.org/wiki/Synchronous_condenser


But with 1500km it would be 90° out of phase and that would be already a pretty high voltage.

I'm also wondering since a couple of years about the same question.


Your issue is seeing 2d (perpendicular) waves where there are actually 1d (parallel) waves. Also, you're not really creating a new wave, you're just giving a "push" to the existing one.


Stations within the same grid are in sync, plus or minus propagation delays. When a newly started up station wants to connect to the grid, it has to get its generators in sync with the rest of the grid before being switched in. (See wikipedia entry for Synchrocope.)

Once the generators are switched in, they'll inherently maintain sync, as even if there's too little motive power being supplied by the station's turbines, the difference will be made up by the power grid itself, which will drive the generators round like an electric motor. (Typically this would happen for a brief period after coming online, after which the station would throttle up and start helping to push the generators round.)


If stations are not in sync, will that mean they cancel eachother out like with soundwaves and anti-sound?


No, it is worse than that.

If you think of two generators connected together in a power grid, they need to have the same frequency and the same phase. If one of the generators slows a little bit, it begins drawing current from the other. It in effect becomes a motor. Power flows as a proportion to the difference in phase angle.


All the other generators in that grid will drive the out-of-sync station like an electric motor at a faster or slower speed until it's synched up. In effect that's what's happening in the article. Unaccounted load is pulling other generators down, making the frequency unexpectedly low.

Perhaps they're not using just frequency to coordinate (solar/wind perhaps needs a sidechannel?), and someone on their grid in a politically unstable area is using power without reporting on the side-channel. Hence the frequency drops due to the unaccounted load.


An old joke says that the power plant starts rotating around the generator.


I imagine that they might cancel out, given limitless current flows between them. In practice, something would go bang long before that happened.


You're missing resistance losses, essentially. They would do no work (no real power component to the load impedance mismatch) but I^2 * R means the complex component would be entirely lost to transmission lines / coil windings.

But the effect is as you say: something goes bang.


Ah, this is very interesting. Thank you.


This propagation delay does not really matter as long as neighboring power plants are synchronized with each other.

Yes, east coast and west coast are out of phase, but only to a perfect observer who can see phase value on both coasts instantly (without a propagation delay). However in reality information about the phase cannot travel faster than speed of light, so to any real observer anywhere between east and west coasts both sides appear to be in sync.


Almost all of the other replies are excellent. In light of your question I think you'll find interesting this realtime map that shows how far out of whack the regional grids are from 60Hz: http://fnetpublic.utk.edu/frequencymap.html


Seems like there's plenty of deviation in North America, more even than the European grid is complaining about. Quebec and the Maritimes seem to oscillate between low and normal, and the west seems to always be a bit too fast.


Yeah you're seeing the short term variation at that site, and there's plenty of it. But over a 24 hour period - for now anyway - NERC standards require time error correction, http://www.nerc.com/files/BAL-004-0.pdf

Going forward it may not be a good idea to rely on the grid's timebase as there's talk as well as an NPRM out there to do away with the standard. https://www.federalregister.gov/documents/2010/03/29/2010-64...


That's a really cool visualization. Do you know if anything like this exists for the European grid?


Not as cool a visualization, but https://www.swissgrid.ch/swissgrid/de/home/reliability/wam.h... shows the frequencies and phase differences (in degrees) between a few measuring stations around Europe. The phase difference between Switzerland and Spain is now some 50°, which is actually much larger than I would imagine these differences becoming.


http://www.netzfrequenz.info

All power grids are connected, so there is just one frequency.


Aren't all the grids in US/Canada also connected? My understanding was that they are all connected and that the grandparent visualization was showing the frequency differences across major portions of the US/Canada grid. Or am I getting this wrong?


No, they are not. There are four major grids: West, East, Quebec and Texas. Each one is referred to as an "interconnection". Synchronism is maintained within each interconnection, and they may have non-synchronous (i.e. DC) links between each other. Here is a map:

http://www.nerc.com/AboutNERC/keyplayers/PublishingImages/NE...


As well as the other answers here, you can control flows around grids by using a special arrangement of transformer to shift the phase of the signal. See: https://en.wikipedia.org/wiki/Quadrature_booster

and here is a commercial example: https://www.siemens.com/global/en/home/products/energy/high-...


Having thought about this before, here's how I think it works. The relevant effect isn't the speed of light per se -- it's the phase velocity v of 60 Hz AC power transmission, which is indeed reasonably close to c. If you have a power line of length l running East to West and the phase on the Western end lags by 2pil/v, then an electromagnetic calculation will show that power is moving Westward [1]. If the western phase leads, then power flows Eastward. If the phase is the same everywhere, then no current or power flows.

This is actually quite handy. When demand locally exceeds supply, the phase will lag (this is how generators work), and power will flow toward the excess demand.

More advanced grid operators use various devices to intentionally shift the phase to control the flow of power. [2]

[1] Actually calculating this is surprisingly complicated. You can integrate the Poynting vector; you can model the inductance and capacitance of the line, calculate the current, and use P=I*V; or you can probably do it in several other ways. You might even be able to model it as little packets of energy moving along at the group velocity.

[2] This ability is important for economic and engineering reasons. Imagine you connect two cities with two parallel, competing transmission lines. One has 100 MVA capacity, and one has 10 MVA capacity, but they have the same impedance. (This is a bit farfetched, but capacity is related to impedance, heat dissipation ability, and the ratings of whatever equipment is at the ends of the lines.) Without some kind of active control, to much power will flow through the 10 MVA line and it will fail.


https://en.wikipedia.org/wiki/Continental_U.S._power_transmi...

> The electrical grid that powers mainland North America is divided into multiple regions. The Eastern Interconnection and the Western Interconnection are the largest. Three other regions include the Texas Interconnection, the Quebec Interconnection, and the Alaska Interconnection. Each region delivers 60 Hz electrical power. The regions are not directly connected or synchronized to each other, but there are some HVDC interconnections.


Yep, as others have pointed out, grid operators can see waves of frequency change propogate through the system.

Here’s some visualizations of that (see the linked movies) from a cool project called FNET/GridEye:

http://fnetpublic.utk.edu/sample_events.html


I love this brain teaser. If you want to make it even more weird, consider that the second plant is communicating it's phase back to the first one! Paradox!

But the answer is contained in the parent comment. Each plant generates more power if the phase lags and produces less if the phase leads. Basically, electricity consumers drag on the phase and the plants use this to control output power, so that the phase matching can be done purely locally.

But it is a lot simpler if you use high voltage DC! If you do that you can connect networks with entirely different phases!


There's splits in the grid:

https://en.wikipedia.org/wiki/Continental_U.S._power_transmi...

So that takes care of some of it.

Anyway, synchronization is actively managed, when a generator starts up, it is synchronized to the grid adaptively.

https://en.wikipedia.org/wiki/Synchronization_(alternating_c...


Even with the splits in the grid, the time lag is still significant, I would think. The western interconnection appears to be 3,700,000 from the corner of Arizona to the corner of British Columbia. The wikipedia article doesn't address the time lag (except perhaps implicitly).


Indian Point to Yankee Nuclear is 230 km, IP to Niagara Falls is 440, and NF to YN is 500 km. At c (electronic energy propagates slighly slower), 1 km = 4.32' (arcminute) of lag. That means the 'legs' of this triangle have lag of 16, 32, and 36 degrees. That around half a Hz of frequency out of phase, or ~0.9%. By comparison, the 2011 tornado outbreak caused 0.09 Hz deviation.

So there must be some accounting for this propagation error, either as losses, or engineering.

What I suspect (totally not my field) is that the high power interconnects between large generators do have this triangular lag (the incoming lines lag from the plant's reference frame), but they use quadrature boosting or similar to match that particular interconnect.

T&D (transmission & distribution) result in ~6% of the net energy being wasted. I imagine phase lag "feels" like impedance, with the real component acting as resistance.

Also, it's worth noting part of the appeal of HVDC, aside from line reactance and skin effect, is you get to choose your output frequency.

https://www.nema.org/Products/Documents/TDEnergyEff.pdf


Reading about this for the first time, it sounds like this problem would also apply to the Synchronous grid of Continental Europe [1] discussed in the press release. This is approx 4000km across, and much more if you include Russia (I'm not clear on the extent of the grid). It does not sound like these are split into individual grids.

Maybe active synchronisation solves this, but to my (inexpert) mind the network would still suffer from interferences at various points between power plants.

[1] https://en.wikipedia.org/wiki/Synchronous_grid_of_Continenta...


> If the USA's power grid is about 4,000,000 meters wide

BTW North America actually has eight transnational (USA/Canada) interconnection authorities or "regions" in which power distribution is managed; it's not a unitary entity.


One interesting thing about the "load changes the frequency" thing is that if you measure the frequency in enough places, you can actually see how large load changes "ripple" through the system.

For example, there was a big power outage in Florida in 2008 that caused a generator to suddenly go offline, and several orgs had a couple dozen power frequency meters running on the grid at the time, so they were able to make an animation of the east coast power grid "ringing" over the course of about 10 seconds as the load changed rapidly throughout the grid.

The animation for that is here: https://www.youtube.com/watch?v=bdBB4byrZ6U


Really interesting to see how some chunks lag more or less than would be indicated by geography alone. For example, the Chicago and Indianapolis areas seem to be much more affected from the northwest than the southeast. I wonder if this is mostly due to topology or just grid capacity.

Do you know how the power plant's control loops are tuned? It seems like just tuning your plant to be properly damped isn't enough, because there's also lots of feedback from other power plants.


I don't know for a fact how they manage it, but I bet the feedback is slow enough (those generators are heavy, and it probably takes a few cycles to even accurately measure a change) that they might not have to do much in the way of actively managing that feedback. The system will oscillate a bit (as it did in that video), but there's nothing inherently wrong with that, as long as it's within a certain set of limits.

(And really, the load changes are typically more gradual. Even "everyone just got home from work" is a fairly spread out event, compared to, say, the sudden loss of a few hundred MW of generating capacity...)


Break time in big sport events also leave there fingerprints in the grid frequency. Everybody start to make a coffee, go to toilet (light), etc.


An example of how frequency is closely monitored before energy demand surge in Britain due to Tea Time:

https://www.youtube.com/watch?v=slDAvewWfrA https://www.geek.com/geek-cetera/tea-time-in-britain-causes-...


Worth noting that TV pickup mostly _isn't_ caused by electric kettles:

> TV pickups [...] are a surge in demand caused by the flushing of toilets (leading to a surge at the pumping stations) and the opening of fridge doors by millions of people. There is a common misconception that the number one driver of TV pickup is the boiling of kettles. In fact, this only creates a pull on the local network for a short period of time until the water has boiled, and can therefore be managed relatively easily, whereas flushing the toilet causes a longer surge at the water and sewerage pumping stations, and opening the refrigerator lets the chilled air escape, causing the compressor to run.

https://en.wikipedia.org/wiki/TV_pickup


Very interesting, thanks. At 1:22 "Power surges like this are unique to Britain. No other country in the world switches on so many kettles in so short a time." I'm not sure if that's true... I did some work for a power company in New Zealand about 14 years ago and this exact phenomenon was known there. Chip off the old block...


I clearly understand far less about electricity grids than I thought. Isn't that 50 Hz frequency generated by the power plants? It seems hard enough to keep them all perfectly in sync. But I don't see how demand can drop that frequency, and even less why you would want to propagate such a drop in frequency across other subnetworks.

Personally I'd expect a shortage to result in a lower voltage, not a lower frequency.

I'm also rather surprised that the correct functioning of my clock depends on the political stability of a politically unstable part of the continent.


You can think of it like the rotor of a generator is spinning at 50Hz under a certain load. If you now reduce this load, the generator experiences less counter force, but its input is still the same, so it starts to spin faster, increasing its frequency to say 50.1Hz.

The opposite is also true, if you suddenly increase the load on the generator, the generator will start spinnging less fast and the frequency drops to say 49.9Hz.

This is all very carefully monitored, and there are multiple automatic (massive) power breakers throughout a power network to detect these changes. Imagine all power lines of a large part or entire country suddenly dropping away. The network outside of that country would suddenly experience a large decrease in load and its frequency would go up, while inside the country the load would have a large increse, reducing the frequency in that country. If the change is so large that the frequency goes above 52.5Hz or below 47.5Hz, then the automatic circuit breakers trigger to prevent damage to the entire network (basically adjusting the load of the network). If it can't adjust enough, more circuit breakers will trip, causing larger outages.

Example of these are the 2003 Italy blackout and 2003 US/Canada Northeast blackout.

Reconnecting a blacked out part to the grid isn't easy either. People tend to leave things on during a blackout, such as airconditioners and TL lights. Now when the power comes back, these things draw a low of power for a short period of time during startup. Normally this isn't bad, but when they all do it at the same time the load is massive (think of a power spikes of 7-8x the normal usage), usually tripping the same huge circuit breaker again.

As for the clock deviation, apparently running below 50Hz for a longer period of time has been condoned by the people in charge of the EU power grid. Maybe for too long. There are two solutions: - generate more power in the EU, but that costs money. They could sell the power to the slackers (which I guess they don't want to pay for) - cut the slackers off (which is very drastic and does not help the EU's idea of looking out for each other).

But a decision has to be made. Looks like more power is currently being generated, as the frequency is now on 50.010Hz.


Imagine all power lines of a large part or entire country suddenly dropping away. The network outside of that country would suddenly experience a large decrease in load and its frequency would go up, while inside the country the load would have a large increse, reducing the frequency in that country. If the change is so large that the frequency goes above 52.5Hz or below 47.5Hz, then the automatic circuit breakers trigger to prevent damage to the entire network (basically adjusting the load of the network). If it can't adjust enough, more circuit breakers will trip, causing larger outages.

If the problem is that the frequency is too high, i.e. power load is too low in that coubtry, why would tripping a break help? Wouldn't that very suddenly aggravate the problem and create even higher frequencies?


When frequency is dangerously high, overfrequency protection relays at generating stations will trip, bringing the system toward balance. When it is dangerously low, loads will be tripped by the under-frequency load shedding system, again bringing the system back toward balance.


This just goes to show that everything is political.


The original article linked to this page[0], which provided a good analogy:

"On a level road it is easy to maintain speed. On reaching a gradient, however, the rider needs to make more effort to achieve the same speed. Going downhill, the rider needs to apply the brakes to keep the same speed.

In the entire European network the electrical generators are set up in such a way that they automatically and immediately respond to a change in grid frequency. Depending on the level of consumption they increase or lower their capacity. This ensures that the frequency remains stable. This automatic adjustment can be compared with the cruise control in a car."

0. https://www.swissgrid.ch/swissgrid/en/home/experts/topics/fr...


Thanks for the correction, as more you read the more interesting it gets.


It definitely does.

And very interesting thoughts come up, especially when you add newer developments like non-rotational-mass-based power sources (solar power, batteries, stuff like that) into the mix. Because at the moment with the high number of traditional steam-powered generators, the rotating mass of the turbines and their inertia serve as a kind of buffer against sudden swings in electricity demand with an ultra-quick response time of "basically no time". As a second buffer stage we have the steam supplies in the plants, which can be quickly accessed just by opening a valve, thus the response time is non-zero, but still quite short.

Now replace all those generators with some other power sources without those inherent balancing capabilities (that probably weren't explicitly built into them, but that just happen to be there because of their construction) and you'll get a network that is much more vulnerable to sudden changes on the demand side. Now, explicit mechanisms of storing energy in a way that can be accessed in literally zero time need to be introduced into the network, just to keep it stable and to provide other regulation mechanisms (of which a lot of them also have to be re-thought) with enough time to do their job.

I wouldn't wonder if there were already plans for building large rotating masses with motors on them that are kept at network frequency, with the motors switching over to serve as generators when the frequency drops.


Synchronous machines with no net shaft power are sometimes used to supply reactive power, but these also contribute a coupled rotating mass. (edit: Apparently English people call these "synchronous condenser")

The question of "how can we build a large and stable grid without thousands of tons of rotating mass?" is an area of very active research I've been told.


Aren't the large Tesla battery farms in Australia being used for that purpose? [1] Basically they use batteries and control systems to correct instead of a rotating mass. I'm not sure about the ability to push several hundreds of MW into the system but at least it gets something done.

[1] http://www.bbc.com/news/world-australia-42190358


The rotor in a synchronous machine is coupled magnetically to the grid (basically a spring-mass system where the spring is a rotary field); there is virtually no reaction time in this mechanism.

(It should be noted that other motors, including asynchronous machines, contribute as well)

AIUI the HPR has reaction times on the order of a few seconds.


HPR reaction time is milliseconds for frequency response. It reacts faster than the grid operator’s data logger resolution.


> I wouldn't wonder if there were already plans for building large rotating masses with motors on them that are kept at network frequency, with the motors switching over to serve as generators when the frequency drops.

Flywheels? Yes: https://en.wikipedia.org/wiki/Flywheel_energy_storage#Grid_e....


Well...that's essentially the pumped-storage model, except it also takes tens of seconds to spool up: http://virtualniprohlidky.cez.cz/cez-dlouhe-strane-aj/


Definitely. Dinorwig[0] has the turbine hall built inside a mountain, using a lake in a disused slate quarry. Stop to maximum power in 12s or something similarly ridiculous. They do tours, which I highly recommend if you are in the UK. You go by bus (inside the mountain) to get to the interesting things.

It was also built to be able to do a black start (startup after a total grid outage).

https://en.wikipedia.org/wiki/Dinorwig_Power_Station


>It was also built to be able to do a black start (startup after a total grid outage).

Aren't all stations?


Nope. They have to be able to cope with extreme changes in load, and I think have higher precision of frequency adjustment to pull the rest of the grid into sync.

Some coal plants had gas turbines[0] fitted solely to allow a black start, others a ton of diesel generators. Some plants need power from the grid to kick off - nuclear for instance. Only some percentage of the grid needs cold start capability, and the rest of the grid is bootstrapped from that.

[0] (edit) Think aero engine. Rolls Royce Avons were used at Didcot A, and the industrial Avon is still available after 60 years. That's the same basic engine found in the Hawker Hunter and EE Lightning.


I recently got to take a look around a small (~ 4MW) hydro station. Despite this being a tiny contributor to the overall grid, it is apparently a grid/regulatory requirement for them to be able to perform a black start.

For that reason, all the equipment in the generator building ran on 48V DC, supported by a large bank of batteries.


Additional anecdata: The sensors and controls in distribution substations are also DC although 48VDC is considered the low end in those places where it goes up to over 200V, according to the engineer from a municipal electric company met in an $OFFICE meeting Monday. The guy said they specify flooded lead acids and plan on them lasting 15 years before pre-planned change-outs.


In addition to the other answers, large generators usually use externally-energised coils to provide their magnetic field, so they can't generate unless there is already a supply of electricity. Then there's all the peripheral machinery of a power plant such as coal crushers and feeders, pumps, valves and so on.

Somewhere there will be a manual for the unfortunate operators who will have to restart the whole grid in the correct order, relying on the telephone company's backup generators to communicate.


Larger stations could take a long time (like a day) to start up, as there are trade-offs between start up time and operating efficiency. During that time, you still have to be able to run all the plant equipment - pumps, fans, coal grinders, conveyors, all the monitoring and control equipment. I expect there will be many stations that don't have anywhere near sufficient diesel generator capacity to allow start up without any supply from the grid.


Probably my favorite anecdote about the grid synchronization behavior is that the generator governor settings (e.g. how to react to changes in frequency) were developed early in the 100 year+ history of the grid, and now we don’t fully understand why the settings were chosen to be what they are, and all the engineers who developed them are long dead.

Now that a bunch of wind and solar is being added to the grid with completely different control schemes, there’s a lot of concern about whether the system will continue to stay reliably synced.


HN-worthy comment. Do we not have any electrical engineering degrees at the universities?


I thought that the frequencies were originally decided by whatever machine the generator was driving. Want your big saw to split logs at 100RPM? Make your water wheel drive a generator at 100Hz. IIRC every big motor was different and ran at its own frequency.

Only when people started interconnecting these generators in a grid did the need to settle on a single frequency arrise.


This is super interesting ! I have a few questions !

How does the demand affect the frequency ? Is there a logical unit, somewhere, responsible of "communicating" through the network ? Or is it a physical effect ?

> When a factory is powered up, the turbines in the power plants are experiencing larger forces attempting to slow them down.

How does something like a higher demand, a drop in frequency, translate to a larger force attempting to slow down the turbines ?


Energy conservation. You're pushing (literally) against the electrical workload of the connected circuits (places where people have plugged a device into a socket and made a loop). When there's more to push against, your steam goes less far and your turbine starts to slow. You can then apply more energy and push the system back up to speed.


The physical effect is known as the Lorentz force: https://en.wikipedia.org/wiki/Lorentz_force

If this was not the case, we could basically generate arbitrary amounts of power from nowhere, as there was no resistance and we could thus just spin a generator forever with zero power necessary (except for the little bit required to overcome friction) once it is set in motion. The Lorentz force makes sure that's not the case by slowing down the movement, and it slows it down more if more electric power is pulled from the generator.


If anyone's thinking of making a clock that assumes the average frequency is 50Hz: Don't do that. There is no commitment to having the average frequency be 50Hz. There is a commitment to keeping frequency close to 50Hz, but no one in the energy industry is thinking about the average. So don't build things on top of that.


That is actually not true for the synchronized european power grid we are talking about here. The time that a clock that runs of the power grid is known as "Netzzeit" (grid time) and there is standards on how far that is allowed to deviate from the legal, public time.

There is websites such as https://www.swissgrid.ch/swissgrid/en/home/experts/topics/fr... that track and display the deviation.


Way back in the dim distant past I had a lovely Sony cube radio alarm clock. It had a switch on the back to determine the clock speed, either 50 or 60Hz. One day, unbeknowst to me, I, or someone else, moved the switch from 50 to 60Hz.

When the alarm went off the following morning, I proceeded to get ready for lectures (This clock was my only means of telling the time, there were no phones or computers in my room) and after having showered I switched on the TV (which could only receive 3 channels at the time!) where I was met with unfamiliar programmes. This confused me for a long while until I realised the time was about 40 minutes later than the clock told me. I missed my first lecture.


It depends where you are. I know for example that the Australian standards require an accumulated time error of less than +/-5 sec on the mainland (+/-15s in Tasmania).

Looking at other comments, it seems that we're a little weird down under and most places don't track the drift.


There is definitely a commitment. European Power Providers keep the average frequency at 50hz in relation to UTC time. With very very high precision.


Yes, there is, any accumulated phase error during the day is normally corrected at night.


Additionally, it will be a region-locked clock by its very nature. We use 60Hz here in Brazil.


When moving from the US to Europe, I had the opposite problem. I had brought along a number of devices that I powered off a transformer (230V -> 115V) here, among them an alarm clock. Which worked off the AC frequency and thus was losing 10 minutes every hour on 50Hz.


Wow, this explains why the clock on my gf's oven had deviated so much after years of keeping perfectly in sync with all the other clocks on our devices.


If anyone is interested in how the frequency and deviation changes are related (i was), i threw this quick-n-dirty chart together this morning: https://docs.google.com/spreadsheets/d/e/2PACX-1vR6ZsthgLiQA...

It scrapes the swissgrid.ch page (with something like "curl | grep | sed > csv" in a loop), so the data is only as accurate/current as theirs…

Edit: here's the actual sheet in case you want to "fork" it – https://docs.google.com/spreadsheets/d/1i9cLeKb5Eq1IoZFOUoOk...


Your answer doesn't seem to address the missing energy, said to be 113GWh, though - which seems the bigger issue according to the report.


Minor nit—a generator is likely to not be rotating 50 times each second. Each rotation of the shaft could be generating multiple cycles depending on the wiring of the generator.


If it worked like that, why would you base your clocks on these cycles?


As a community of mostly engineering-minded folks, I hope we can take this as a warning to not make our mission-critical systems depend on inputs that are merely convenient side-effects of third-party systems provided free of charge and without obligation. I mean, it doesn't matter how many people post comments here saying things like "50 Hz AC is more accurate than quartz crystal for timekeeping", the fact is that no external agency can take over the action of your quartz crystal and force your clocks to slow down or speed up, whether intentional or not, as an effect of their political actions.

If you want a free source of mostly-reliable oscillations, and it doesn't have any real impact if it falls out of sync, sure go ahead and use something like this. But in the case of time, if you want to sync a clock in Western Europe with something that is free and non-internet based, you're much better off using DCF77 [0], which is specifically designed to synchronize time and comes with specific uptime and quality promises. Still ultimately susceptible to political actions, of course, in extreme cases, but at least you know its primary purpose is to transmit time information, and it is in the control of only one (relatively) stable government as opposed to being subject to the unpredictable changes brought about by interactions between multiple interconnected systems.

DFC77 doesn't solve the same problem as using AC as an oscillator, of course, since you still need an oscillator to keep your clock going during the downtimes that are permitted to the radio signal.

[0] https://en.wikipedia.org/wiki/DCF77


>depend on inputs that are merely convenient side-effects of third-party systems provided free of charge and without obligation.

I would disagree. You don't get it for free, you pay for that. The Grid frequency is regulated and the powergrid providers have, IIRC, even legal obligations on how far they are allowed to deviate the time.

Using AC as clock signal is good enough if being wrong by a few minutes is not mission critical. If you absolutely do need accurate time use DCF77 or GPS.


> As a community of mostly engineering-minded folks, I hope we can take this as a warning to not make our mission-critical systems depend on inputs that are merely convenient side-effects of third-party systems provided free of charge and without obligation.

That is not entirely true. The TSO is committed to keeping the average frequency of the power grid at 50Hz and has always communicated that. It tries to keep grid time within 20 seconds of UTC, and has historically achieved this. So it is reasonable to use the grid as a time source where high accuracy is not required. The current situation is highly irregular. Using the grid frequency for time keeping is a secondary function of the electricity grid, but it is not merely an unintended side-effect.

I agree with the rest of your points about using a different time source where high accuracy is required.


Good point about this service, I almost forgot that it still exists (although reception is somewhat spotty indoors). The basic advice is sane ("do not rely on side effects"), yet "third-party systems provided free of charge and without obligation" sounds exactly like that, except now it's putting all your eggs in a basket that's controlled by one entity (be it DCF77, or the GPS temporal component). I would think that the AC-as-a-clock is usually chosen for convenience rather than high reliability - "the juice is needed anyway, why bother with an extra receiver?" - and if you actually need to tell the time, checking multiple independent sources would be encouraged :)


DCF77 surely cannot be that bad indoors on the Continent?

When living at what was then often quoted as the extreme fringe of the coverage area - Trondheim, Norway, on 63,5 degrees of latitude - I had reliable coverage as long as the clocks were kept in window sills (hence, effectively outdoors)

However, just moving down to my current home (on 62 degrees, or -roughly speaking- 11% closer to the transmitter site), I now have reliable coverage everywhere (granted, in a wooden house - but the DCF77 alarm clock in the basement synchs every hour, too)


I've found that antenna orientation matters, even in the Netherlands which is much closer to DCF77 (concrete house).


Citizen, the Japanese watches maker, offered a passive (without batteries) "enhancer" for their AT series, IIRC was a ferrite block where to put the watch, I think I've seen a photo.

Or one can use a PC to transmit the DCF77 signal locally (it could be illegal) to a clock using a pair of headphones, I've seen at least a couple of examples but never tried them.

Edit: found the photo http://forums.watchuseek.com/f17/possible-method-improve-ato...


Yes, it needs to be perpendicular to the direction of frankfurt for best results

You can find more details in the second page of this datasheet

https://www.gemischtwaren-haendler.de/shopdateien/3942_1.pdf


More importantly, you would need to know how the antenna is oriented within the enclosure of your clock. I'm pretty sure that that wasn't mentioned in the manual of the one I bought.


Most clocks are thin, so it's easier to guess the placement of the antenna.


Usually the antenna is parallel to the board - and the board is positioned in the largest dimension, which tends to be also parallel with the display.


Beats me - I'm within reasonable driving distance to the antenna (one border away), yet clocks fail to acquire signal unless outdoors/directly at a window. Just a bad coverage area, I guess.


>11% closer to the transmitter site

This is completely outside of my area of expertise, but doesn't the radio signal strength decrease exponentially? So that last 11% could be a big deal?


Inverse square, so you're in the vicinity of being correct: https://en.wikipedia.org/wiki/Inverse-square_law


Quadratically.

Line-of-sight attenuation of an EM wave in the empty space is quadratical, whilst - for example - attenuation of an electrical signal in a wire is exponential.

Obviously this is a super simplification...


Oh, that makes sense. Thanks for your explanation, good enough for now :)


Another advantage of DCF77 and similar is that they also broadcast the current time, while the 50Hz or 60Hz AC is just a frequency. If you have only a frequency, you have to set the initial time manually every time you lose power.


DCF77 also tells you what time it is, AC can only keep whatever time's been set in sync. The equivalent would be to include a 50-cent crystal oscillator instead.

On a only barely related note, I've been playing with the idea of implementing a NTP synced clock on an ESP8266. It'd wake up, join wifi and sync NTP every few hours, then keep time on its internal crystal in between. It should be a good deal cheaper, work anywhere in the world, and have better indoor coverage (I have a DCF77 clock in an interior bathroom in London, and it never syncs. Kind-of defeats the purpose that I have to move it to a south-facing room for the day to get it to sync).


The UI for entering the Wifi password is always the biggest problem with these things. The Dash buttons have an interesting solution involving audio.


You could try a USB SATA drive with a FAT16 filesystem containing "/wifipass.txt". USB-1.0 low speed is 1.5Mb/s and while I can't find documentation on ESP8266's GPIO switch speed, it's CPU is 40MHz, so bitbanging should be feasible.


One solution would be qr scanner...


Or just reporpose an old phone.


I was thinking about this in terms of building a product.

Even with that, a clock generally runs for months, even years, on a single battery. Not exactly phone territory, but might be feasible on the ESP8266, depending on how the oscillator behaves in deep sleep mode.


There are many dedicated real time clock chips, pre-designed for backup time-keeping and low power usage. Interfacing with one is going to be way faster/easier/better than trying to roll your own on a general purpose microcontroller, IMO.

TI's bq32000 is $0.55@1K, is 3.3V for easy interface to the ESP, and takes just over 1 microamp in backup power mode. That's just one that I quickly poked at.

(Side note: at this point, I'd think I'd want to be using an ESP32 in any new designs.)


>how the oscillator behaves in deep sleep mode.

horribly


I imagine the difference between DCF77 and a 50Hz clock is similar to the difference between a $40 clock and a $10 clock. :P


Radio-controlled is pretty much the default around here (and the cheap ones cost perhaps 5 €); I think I've yet to see a grid-powered alarm clock.


Here's an IC used in some alarm clocks that uses the power grid's frequency, along with a list of products that use it: https://www.radiomuseum.org/tubes/tube_lm8560.html


It was mostly used in the common radio alarm clocks, the ones with a 4digit red 7 segment display that were all based on an IC from the 70s that handles everything (minus the radio which is a completely indipendent module).


GPS is also an extremely accurate source of time, although since it's on a much shorter wavelength your reciever needs a clear view of the sky and is much more complicated.


My favourite story about grid frequency involves Laurence Hammond, who would go on to create the Hammond organ. He was a designer of AC electric motors, and when he formed his own company, among their first products were luxury AC-powered clocks. The trouble was that they didn't keep very good time, because of fluctuations in the grid frequency.

He solved this problem by making gifts of his clocks to executives of the local power companies, and as if by magic, the timekeeping of all his clocks soon improved.


This map of the European power system is linked from The Featured Article: https://www.entsoe.eu/Documents/Publications/maps/2017/Map_C... [complex pdf, doesn't render well in some viewers].

Several fascinating points:

- Denmark is split between the Nordic (Sjaelland and Lolland) and continental systems. Anyone know why?

- Even North Africa is synced to Europe.

- A corner of northeast Poland is fed by Belarus.

- Cyprus is disconnected, and northern Cyprus apparently has no network to speak of.

- You can see the DC links to the other systems, and e.g. a back-to-back converter at Alytus, Lithuania, that isolates the European from the ex-Soviet system. But I see no such arrangements at the Belarusian and Ukrainian borders, or at the borders with Syria and Iraq. Is this just missing information, or are these countries also synced in some way?


I worked on software for the Danish energy grid 10 years ago, so my information may be a little outdated. There is actually a connection now, or at least it's planned, but the grid was split simply due to historical reasons.

The Jylland-Fyn grid and Sjælland grid was developed in parallel so we ended up with a grid containing 400kV and 50kV cables West of Store Bælt and 60kV East of Store Bælt.


Your comment prompted me to do some research on that line to Poland.

According to sources I've found (all in Polish, so I'm not linking them here), this 220kV line Between Bialostok and Rossj was built in 1962 and is out of use since 2004. The power to the region is delivered by 100kV lines which are not shows on this map. There was an idea to build a new one double 400kV line to import energy from Belarus but nothing going on so far.


Nice map. Now I understand why TeNNeT (the Dutch semi-state run power transmission/line operator) owns part of and is investing in north Germany since the wind fields basically run directly into the Dutch power grid.

North Cyprus does not surprise me but Crete apparently has no power distribution to speak of or only 50kV lines for transmission.


There's a big ugly gas power station on the north coast of Crete on the edge of Heraklion. I stayed in the hotel next door :-/. There's quite a lot of solar on hotel and house roofs too.

From memory they were planning to have a subsea interconnect installed.


The Great Belt Power Link (DC) in Denmark wasn't built until 2009/2010 to simply reduce price variability between east and west. Sjælland is close to Sweden and the rest of Denmark is easily connected to Germany, so early interlinks made the split naturally.


For some reason I'm having trouble opening the PDF, both in Firefox and with evince on Linux.


This is really heavy pdf. Try llpp or mupdf.


Weirdly, it opens fine (but slowly) in iOS Safari...


mupdf worked like a charm! Any idea what makes it so performant (or evince so little)?


mupdf is, in my experience, much faster than poppler for complex documents, but mupdf-based viewers generally offer less features than those employing poppler's services (like Okular). These two things might be related.


Okular will do it, just takes a minute to do so. There are a huge number of layers to the PDF so it's rendering a lot of detail.


I could only open it with Gimp (which rasters it to a bitmap) on Ubuntu with the installed default software.


This one [1] was linked on another blog in Germany on a discussion of the same topic, seems more like a Google Maps type application and is not as heavy to load (also misses some legend, but... well).

[1] https://openinframap.org/#4/47.96/21.01/Power


> A corner of northeast Poland is fed by Belarus.

You mean Kaliningrad? Because that is not Poland but a Russian exclave


Kaliningrad oblast is clearly labelled "Russia" on the map so it seems unlikely that anybody would make the mistake you're implying.

Looking at the map it's clear that OP means Białystok.


Actaully, after having read the legend at the left of the image, it is clear that many lower-power lines are left out, so it is possible, even likely, that Białystok has a connection to the rest of the country.


Ukraine and Moldova are in process of joining ENTSO-E.


Trying to find more details on the specific political issue (ie, not the regional baseline, but the power issue in particular).

Best I've got is an offhand sentence that Kosovo is using more power than it produces and Serbia refuses to balance that consumption.

https://www.platts.com/latest-news/electric-power/london/kos...


I found some more information: http://www.balkaninsight.com/en/article/government-pays-a-mi...

Since the end of the Kosovo war in 1999, the four northern Serb-majority municipalities have not paid Pristina for their energy consumption.

To make up for the shortfall, people from other areas of Kosovo had a percentage added to their bills to pay for the north’s electricity.

In December, the Energy Regulator’s Office announced that electricity bills will be reduced by 3.5 per cent as consumers will no more cover the cost of the four municipalities’ power as they have done for the past 19 years.


I imagine everyone was Bitcoin mining there and the bill was going thru the roof.


Would you believe me if I told you that because of the legal situation (Northern Kosovo does not pay any electricity), there have been tons of crypto mining operations that have sprung up. There are stories of Russians coming all the way to set up warehouses full of mining rigs and generate cryptocurrency using free electricity.

Source: I live here.


also (speculation):

raising this issue now (rather than earlier in the year) in response to the near-end of the syrian pipeline proxy war, bulgarias mysteriously financed repurchase of it's own gas lines from czech holding co, lavrov and gazprom ceo in belgrade unveiling giant mosaic, etc etc etc...


Thank you for this and the child comments, I've been looking for the tl;dr of the political explanation. While the discussions of network frequencies, generation, phases, and propagation were all very educational, HN couldn't see the forest for the trees. Fascinating that the bitcoin mining (an HN favorite) was involved.


Wow, I didn't know continental Europe is a single synchronized area. Here in the Nordics (Finland) our grid is separated from the neighbours via DC stations to eliminate the need to synchronize with everyone. Here you can see our grid info: https://www.fingrid.fi/sahkomarkkinat/sahkojarjestelman-tila... - "Sähköverkon aikapoikkeama" is actually the cumulative difference between the 50Hz reference and the actual frequency in the network. When that stays zero, AC clocks keep their time.

edit: I stand corrected, we are in the same frequency domain with Sweden and Norway. I must have confused myself with the DC submarine cables under the Baltic.


Eliminating the need for synchronization isn't the only benefit of high tension DC power lines. DC is also sometimes used for long-distance transmission lines to avoid hysteresis losses. The downside to DC is that voltage step-up and step-down equipment is more complicated/expensive and (at least generally) less efficient.

In high school, I was told that a lot of the power for Minneapolis was sent over high tension DC lines from coal power plants in the neighboring Dakotas for this reason. This was part of the lesson about how the lakes in northern Minnesota had very little pH buffering capacity due to limestone being scraped down to bedrock by glaciers and therefore being particularly sensitive to SO2 emissions from coal power plants in the Dakotas, but most of the electrical demand for that power also coming from Minnesota.


That's interesting! According to this map [0] the AC connections are only between Sweden, Norway, Finland and a large part of Denmark. At least under the assumtion that the frequencies are separated in the "converter station back-to-back" in Vyborgskaya, Russia.

[0] https://www.entsoe.eu/Documents/Publications/maps/2017/Map_N...


"Vyborgskaya" sounds like "station of Vyborg" and not a separate toponym.



French grid data from RTE is available from http://www.gridwatch.templar.co.uk/france/

I took the frequency data from the last 1200 days, and calculated the cumulative drift. The raw drift graph, assuming 50Hz is nominal, is here:

http://nrg.cs.ucl.ac.uk/mjh/drift-raw.png

This seems bogus; there's a constant drift.

If I compensate for that drift by assuming the nominal grid frequency is 49.9972Hz, the data looks a little more sane.

http://nrg.cs.ucl.ac.uk/mjh/drift.png

You can clearly see the 6 minute deviation this year - it's pretty striking, and certainly nothing this drastic has happened in the last three years. But what's with the positive offset over the previous year? Not entirely sure how much I trust the RTE data.


For what it's worth, I have an alarm clock connected to the French grid, and until last month it's been perfectly on time, so assuming it is tuned to 50Hz there is probably something wrong with the source data.


Agreed - without knowing more about the data provenance, all I can show here are deviations from the long-term trend. It's enough to see the recent problem clearly, but I wouldn't trust the increase after day 900 to be real. Most likely something changed in how they report the data around that time.


Awesome work, thanks!


I am almost 100% sure that this is, in part, caused by the recent crypto mining surge. Or at the very least was a catalyst for something that's been cooking for 20 years now.

I have no evidence to back this claim, only anecdotal info as I know some people in the area: The Serbian minority in the north does not pay for electricity in Kosovo due to a long standing disagreement with Pristina. The Kosovo government so far managed to find ways to compensate for that, either through loans or by charging more people in other areas.

However, I'm guessing, the bills skyrocketed when the crypto mining craze started so Pristina decided to stop footing the bill.

I've heard of thousands of mining rigs being set up in the area, largely due to the free electricity. So now there's no more free electricity but there is a huge demand and nobody is paying. Hence the deviations.

http://www.balkaninsight.com/en/article/government-pays-a-mi...

edit: wording


Two days ago, I noticed that my alarm clock and the oven clock where both about 5 minutes late and I was about to create some conspiracy theory about someone manipulating the radio controlled clocks as 5 minutes deviation in the same direction was too much of a pattern to be a coincidence. But then I noticed that my wrist watch, which I hadn't changed since 3 years, still had the correct time. So I doubt my own pattern recognition abilities and changed the clocks.

When I read about the power shortage today, I was kinda relieved to get the confirmation, that my own pattern recognition was very fine, but that my theories where just not elaborate enough :-D


Wow, I had the same! Alarm and oven clocks were off by five minutes from my phone. How strange.


I was curious how the "normal" P=IV formula you learn in school worked here, since I wasn't understanding how frequency could possibly change the amount of power outputted. The linked article (https://www.swissgrid.ch/swissgrid/en/home/experts/topics/fr...) explains the reason: a decrease in frequency means that the generators are actually turning slower, so there's less power generated overall.


A clear analogy is cycling. If you meet more resistance, you get a decrease in frequency. You go a tiny bit slower. So you have to add power to get back up to speed. If not already clear: resistance is the load (wind or going uphil) of electricity consumers to the power generator (your legs).

If you are on a tandem bike, all occupants notice the decrease in speed and add power to get up to speed.


> If you are on a tandem bike, all occupants notice the decrease in speed and add power to get up to speed.

Unless one of them doesn't, and then you have to add even more power to also carry their feet through the rotation...


> A clear analogy is cycling.

Yeah, that swissgrid website linked in the parent comment even has a "Everyday comparison with a bicycle" section. :)


The real power for AC is averaged over the cycle. So the current decreases as frequency decreases.


There is relatively little total stored energy in the rotating kinetic energy of the grid's electrical machines, typically only a few seconds of rated power or so. From fastest to slowest, the response time of the grid to a load change is:

- Stored capacitance of the distribution lines (sub-ms response time).

- Stored kinetic energy of the spinning machines, both generators and loads (sub-cycle response time).

- Automatic throttle management (sub-second response time).

- Spot market for electricity (intervals vary, but 15 min is typical).

- Futures market for electricity (up to a year in advance).

Not all suppliers bid for the frequency control market. Those that do are paid to adjust their throttle back and forth automatically, providing what's known as primary frequency reserve. Normally, every hydrocarbon power plant is bidding for primary frequency reserve. Typically all of the supplier bids for for short-term frequency control have the same ramp rate for automatic throttle management, set to provide a 100% power step per 5% frequency deviation. It makes up the difference between what the spot market cleared and what customers actually demanded during the interval.

Normally, (in deregulated electric markets in the US anyway), a frequency decrease translates directly into a higher price for power, which is cleared by the spot market to return frequency back to 60.0 Hz. A sustained frequency decrease, combined with a net sink of power into this particular geopolitical region, is caused by suppliers in that region failing to provide enough power to clear the spot markets. The difference is being made up by the PFR suppliers throughout the rest of the grid.


Also remember that P=IV is the equation for DC, not AC.


Almost. I assume you know, but P=IV is only an approximation when reactive loads are close to zero. It is not the "DC formula".

S is "complex power", meaning it includes both the real (resistive) and imaginary (reactive) parts. It is measured in volt-ampere, and is calculated as S=I_z x V, where I_z is the impedance current ("complex current").

P is "real power" (resistive) measured in watts, and is calculated as P=I_r x V or P=S x cos(φ), where I_r is the resistive current ("real current"), S is the complex power, and φ is the phase angle or "power factor"—the delay between voltage and current as an angle.

In a pure DC system (think "incandescent bulb on a battery"), the phase angle φ is 0 making P equal S, as cos(0) equals 1. However, in real life, it is only a vague approximation. Electronics switch currents and have reactive components, giving them a non-zero phase angle. They're more complicated to calculate on than an pure sine-wave AC system, not less.

(I apologize for any hiccups above. I stopped being an electrician a looooong time ago.)


Yeah, yeah, you now have a couple √2's because you're dealing with RMS voltages and currents, and a cos(θ) for the phase angle, but when it's all combined it ends up differing by some constant factor.


Well, no, not if the voltage for example doesn't stay constant nor changes with a constant period, because then P=IV becomes an integral that doesn't simplify like we are used to.


I was under the impression that voltage was mostly constant most of the time, except for brief periods where it was switched to keep up with/scale back to match demand. Assuming constant voltage should be a good approximation in this case, right?


Imagine a stone being lifted. The weight of the stone is the current, and the height it is lifted to is the voltage.

Both the weight of the stone and the height have an impact on the energy required to do the lifting.

Now imagine you are lifting and lowering it repeatedly. It should be intuitive that doing so 50 times a second requires more power than doing it 25 times a second (One being more lifts than the other, in the same time period).


Things are a bit more complicated than this. In your toy system a stone that stays still (DC) would have no power (potential energy), which is false. Also in your toy system m is constant. In general for AC you have both V and I oscillating.

As a matter of fact the average DC power does not depend on frequency.

You could posit a system where g is alternating between positive and negative values and the h reacts to it (phase-shifted). You'd quickly come the conclusion that the potential energy (averaged over a period of oscillation) does NOT depend on the frequency. (The kinetic energy does, but that's where your analogy breaks).


> In your toy system a stone that stays still (DC) would have no power (potential energy).

This is false. In the toy system, weight is current, height is voltage. A stone that stays still has constant voltage, not zero voltage. Thus, it would have a 'power'.

What the system lacks is to define the stone as a capacitive load. Then it would sorta make sense.

It is a hypothetical system, so you can only reason about the aspects the author defined. Tying potential energy in the toy system to real-world potential energy doesn't work.

(Btw, potential energy is not power, it is work. Power is work over time.)

> As a matter of fact the average DC power does not depend on frequency.

Uhm. "DC power" stops existing if the frequency ≠ 0, so in that sense it does depend on frequency.

It's true that power itself is not frequency dependent. However, any load is, as reactive losses (parasitic or not) are a function of the frequency. As the power is a function of the load, power ends up being directly tied to the frequency.

(A resistive load cannot exist outside of a perfect DC system, so reactive loads will exist).


Do you mean weight or mass? Is gravity a thing here?

If you define mass/inertia as capacitance, then the resistance to changes make sense. A pure resistive load (which cannot exist, but lets ignore that) is not frequency dependent.


I'll admit I didn't really think this analogy all the way through.


Mmmm, no in alternating current V is alternating as well as I


Yes, but they should both be alternating in a periodic fashion, right?


The simplification for calculating the power of an alternating current by just using the maximal values and a correction factor is only valid if the signal is alternating in a periodic fashion, which includes constant period. However, the reasons this thread exists is that the electricity power signal does not have a constant period, and is therefore not strictly periodic. Then, the simplification for the integral does not hold anymore, and the integral must be evaluated with some other technique.


It is, if V is the RMS equivalent and the power factor is zero


I find stuff like this extremely interesting.

It's incredible how connected and interdependent we have become as a society and how we only notice this if things start to break (or at least run out of spec); even if it only impacts such supposedly minor things as a clock on a baking oven.


I'd love to see a gargantuan DOT file mapping out all the world's supply chains, across all industries and economies. I wonder how many 'only one source in the world' (similar to the 1970's chip saws[1]) lynchpin nodes there are...

[1] https://www.theregister.co.uk/2011/03/22/japan_quake_semi_is...


Here's something similar for >110kV powerlines:

https://openinframap.org/#4/47.96/21.01/Power


A lot of modern society depends on someone not fucking up. DNS could break if the root nameservers stop working for whatever reason. BGP is a major problem if somebody gets a free pass on sending garbage.

Using AC as a clock is not the worst and most of the time extremely reliable.


> The missing energy amounts currently to 113 GWh. The question of who will compensate for this loss has to be answered.

This seems to be the primary issue, not the minor drift in cheap clocks.

Also, the press release says nothing about "energy war" so that kind of hyperbole does not add anything of value.

What is going on here is akin to someone tampering with their electrical meter, except done by a state electrical company, so on a somewhat grander scale.

1 GWh = 1,000,000 KWh and in Serbia/Kosovo the price is around €0.07/KWh so the retail amount of this theft is around €8 million. It is significant enough to make an issue of it, but not really a big deal.

http://ec.europa.eu/eurostat/statistics-explained/index.php/...


The UK's http://gridwatch.templar.co.uk/ keeps track of frequency, although not cumulative drift. The UK is DC-coupled to the rest of Europe.


http://gridwatch.co.uk/frequency shows a monthly average of 49.999 for February for the UK National Grid


I checked for France : average frequency is 49,99456201 Hz in February with one measurement every 10 seconds (source : http://clients.rte-france.com/lang/fr/visiteurs/vie/vie_freq... ).

Same info for November 2014 : 49,99994706 Hz.


I was just yesterday changing my oven clock. Interesting to find out why here on HN. I wonder what critical systems (besides my oven) will start misbehaving. Traffic lights, maybe?

(edit:Apparently, the emergency broadcast system.)


A lot of electricity meters can take have the 50Hz mains as a clock-source...

Leads to interesting questions such as how the (time-based) metering is affected in this situation.


Sure it is time-based? What I've seen is that the meter sits right on the power line, and can measure the energy consumption quite directly. As for peak/off-peak, this is usually signalled by the provider, using AC as a carrier wave, e.g. https://en.wikipedia.org/wiki/Zellweger_off-peak


yes, the meter sits on the power line. Thi sis referring to the clock-source for the metering functionality. In some meters, this is derived from the mains frequency (usually as a fallback tho).


Older traffic lights were controlled by mechanical sequencers: a synchronous motor turns a drum with pins on it (like in a music box) and the pins open or close contacts which drive contactors powering the individual lights.

Modern ones are networked SPSes or small industrial PCs.


I assume all mission-critical clocks get their tick from a quartz crystal. Clocks running on mains frequency is a very cheap cost cutting measure.


If it's mission critical then -- in Europe -- it gets the time from DCF77. It uses an atomic clock to emit a time signal (actually, it uses three and only uses their output when two out of three agrees). It is also compared to further atomic clocks and corrected as necessary.

Even many consumer grade clocks will sync to the radio signal once a day and use a quartz otherwise.


I suppose that in the case where the mission itself is critical as well, then indeed it's going to have an atomic clock signal.


If your clock is mission-critical, it should be rather well kept. Quartz, NTP, GPS, the list goes on and multiplies like rabbits (redundant this redundantly connected to redundant thats and backed up by those...). But indeed, sometimes you don't need sub-second precision, and relying on The Grid is useful.


I only have some experience with lab clock sources, but even those have some amount of redundancy by nature. They are generally a very good oscillator (e.g. a very good OCXO) disciplined to a reference with much better long-term deviation (e.g. DCF77 [uses a PM carrier, not the timing signal], GPS, a rubidium source or a miniature caesium source). Short-term (minutes to hours) is governed by the oscillator, and all these sources will enter holdover when the reference does something stupid. More recent ones can tell you the error margin during holdover. Distribution amps can have redundant and cross-checked inputs for independent sources.


I have one locally sourced answer to that question: our emergency sirens failed their monthly test run because their internal clocks were out of sync!

Source (in Dutch):

https://www.nu.nl/binnenland/5164385/


Ah now I understand why the clock on our microwave oven is running behind.

I am sure I set it correctly after the end of the latest daylight savings period (which they also should forbit, different subject).

It seems getting worse over time. It now runs behind for guess what.. 6 minutes ;)


"Clocks in Europe that depend on the grid frequency" which usually means alarm clocks.

I guess a lot of clocks use a crystal now so they're less dependent on the grid frequency.

Also don't buy an alarm clock in the US and bring it to Europe and vice versa, unless it is user adjustable (or you are comfortable with a soldering iron)


I preferred the old title. It felt fantastically cyberpunk.



We've updated the title from “Clocks in Europe are slowly deviating because of an energy war in the Balkans” to that of the article.


The delay is now up to 6 minutes since January !


Correct. This is crazy. Everything from my central heating system, kitchen appliances and bed-side alarm clock is off by 6 minutes.


You must have a really old alarm clock. And why wouldn't you correct the time manually if you know it's drifting?


What innovations in alarm clock tech am I missing out on that would make me want to upgrade?


Being part of an IoT botnet seems to be all the rage nowadays.


Essentially the whole "watches you while you sleep [not creepy at all] so it can wake you when you're not in a deep sleep phase." I do use a smartphone for that, as it already has motion sensors, and it is indeed somewhat gentler to wake up to. (OTOH, if I absolutely positively need to wake up at $TIME, the ticking mechanical bedside clock of my grandfather it is: it hasn't been innovated for a century, needs to be manually wound, is completely independent of any external source, and rings loud enough to rouse the dead)



A quartz crystal?


6 minutes isn't a whole lot for a bedside alarm clock. I probably wouldn't correct it unless it reached ten minutes or more, which is when it would start making me late for things.


Hmm, I'm seeing 5 minutes 46 seconds, with a frequency of 50.02 Hz. Maybe they're fixing the issue?


Only device I have is my oven that uses this kind of clock... but usually only winter/summer time alters this irrelevant to begin with. Not the mention a power outage sometimes and then months before it's set again.

Then again, what kind of more dependent systems or companies would have negative effects due to this? Besides bedside alarm clocks.


Heaters and similar high-draw industrial automation rely on a clock (which is not necessarily this clock - plus optionally another signal sent by the power provider over AC) to use off-peak (i.e. cheaper) power. Six minutes' running at peak power rates can get expensive at scale.


My microwave adds a minute every week or so because the UK's frequency is slightly different to the the rest of the EU. The clock on the microwave I have is keyed to the continental europe's frequency rather than the UK.

It's actually moderately annoying.

It was a fairly well reviewed (cheap) machine from Wilkinsons, a budget brand. Perils of cheap electronics.

EDIT: Well sounds like it's not the case, TIL, not sure where I picked that up from.


The frequency is exactly the same 50hz.

The voltage is usually 240 volts rather than 230, but within the acceptable range (which can be as high as 256).

Higher voltage is good. The kettle boils faster.


To see what the Grid's up to in roughly real time, I love this site.

http://www.gridwatch.templar.co.uk

After last week where demand was very heavy and frequency was below 50Hz, the frequency is currently the highest I have ever seen (50.107Hz)

You can also see the French grid using the same site. This shows the European frequency at 50.024Hz, so perhaps corrective action is being taken.

Edit - as you were, 5 minutes later UK frequency is 49.902Hz and European frequency is 49.993Hz


You sure that's the cause? The UK has legislation limiting amount of drift from 50Hz, and the Grid adjusts frequency multiple times a day, outside spikes, to ensure the cycle count is correct.


> the UK's frequency is slightly different to the the rest of the EU

It is? What does the UK use?


It's not. His microwave must be faulty.


If we go by his numbers and assume the EU uses exactly 50Hz, then +1 minute over a week (7 * 24 * 60 min = 10,080 min) would mean that the frequency in the UK is roughly 10081/10080 * 50 ~ 50.005Hz.

I'm not sure if that is the case.


It's interesting that fixing this delay will involve setting the grid's frequency above 50 to 50.01 Hz which will advance clocks about 15 seconds a day and thus it will take 3 to 4 weeks to bring the clocks back on time.

And they cannot set it higher than 50.01 because at that point mechanisms for bringing the frequency down to 50 would automatically kick in.


Where does that 50.01 number come from? For the last 25 minutes, the UK grid figures have been:

50.217999 50.049999 49.990002 50.016998 50.106998 50.091999

They seem to go well above 50.01 for significant periods. Data from http://www.gridwatch.templar.co.uk/


The UK is not part of the Continental European Power System.

For the Continental European grid, the article links to https://www.swissgrid.ch/swissgrid/en/home/experts/topics/fr... which states that in the case of a time drift of more than 20 seconds the frequency will be shifted to 49.990 or 50.010 to correct it.


Would it "fix" it though? I mean I've already heard a lot of people that just reset the time on their clocks, so in 4 weeks they have to dial them back again. Maybe for unattended time keeping things?


Most of the clocks do not have a human nearby to push the reset button (if there is one), as they're industrial automation. Alarm clocks and microwave ovens are mostly a side effect here; and yes, if you have adjusted the time manually, you'll need to do that again.


I wonder how those industrial automation clocks deal with power outages. Europe has quite reliable power, but even the best electricity providers have occasional unplanned outages.

As of a few years ago the best was Germany, where the average customer had about 15 minutes per year without power due to unplanned outages (excluding outages caused by exceptional events). (For comparison, the best quartile of utilities in the US averaged about 90 minutes/year for their average customer).


using AC frequency for timekeeping seems ridiculous. Does quartz generator cost anything?


Depends if you're making an analogue clock or a digital one.

If you look inside a cheap mechanical timeswitch [1], you'll find a synchronous motor the size of a sugar cube, turning 50 or 60 times a second, then half a dozen plastic gears reducing the rotation to 1 turn per day. AC wall clocks follow the same principle, but with a slightly different gear ratio.

This means you don't need a printed circuit board, or any integrated circuits, or anything like that. Such efficiency is how you make a profit, if you're selling timers on Aliexpress for $3.50.

If you're making a digital timeswitch, a quartz crystal might be cheaper, or might not - it depends on the insulation/isolation you think your product requires.

[1] http://swarfer.co.za/rc/03-timer.php


You'd expect even a fairly good quartz to deviate by more than a minute a year from temperature fluctuations. Relying on AC frequency should allow your clock to operate for many years without gaining or losing time as long as the AC frequency is compensated, which it needs to be or someone is stealing energy.


In ordinary circumstances, the grid frequency is far more accurate than a cheap quartz crystal oscillator.


What for, if you already have a clock signal that has been reliable until now?


Because its reliability is happenstance.


Nope. There's man-years of effort per day in keeping it reliable; as evidenced by the point that it takes major political meddling over a prolonged period to throw it off, and an article on HN for people to even notice "it has been acting sort of weird, now that you mention it".

"I didn't notice until now" is quite the opposite of happenstance, on par with the PHB mantra "what do we need devops for, anyway, the network seems to be up!" Well it's up because devops is keeping it up. Likewise, the grid doesn't just magically self-balance around 50 Hz.


I definitely noticed this in the last few weeks already, since my alarm clock is set to switch the radio on in the half minute before the news on the top of the hour. I was already searching for reasons, and I would have expected an announcement about this a lot sooner.


I mean, in software engineering we try to avoid unnecessary dependencies right? This is exactly what's happening, why would those clocks ever depend on the power grid??

EDIT: Nevermind, sounds like there are precision concerns that justify using the power grid for timekeeping.


Indeed. There's a tradeoff in both directions - either you trust the cloud^W grid, or you trust your local clock, and both of them have failure modes; there's no one clearly superior option here. You could combine them, but then it gets complex in short order.


> I mean, in software engineering we try to avoid unnecessary dependencies right?

I/you wish.


> in software engineering we try to avoid unnecessary dependencies

leftpad would like to have a word


Putting Lego blocks together is engineering now?

Edit: NPM has engineering challenges, and Node as well...but I would be extremely hesitant to call a library of ~ten LoC "engineering."


i'm not a fan of what the JS ecosystem looks like, but you can and will get bitten by transitive dependencies if you don't store them all in a local mirror that doesn't respect upstream deletions...


Of course. But that's an engineering problem worthy of the name (cache-invalidation and whatnot).


Grids ensure the cycle count is correct over the day. There will be some drift from spikes and peak loads, but they adjust the frequency to compensate. They have atomic clocks, for the EU grid it's in Switzerland, to track the cycle count.

UK and EU grids have websites letting you look at frequency history.

There's still an awful lot of infrastructure keyed off having an accurate synchronous time, including billing systems, traffic lights, street lights...


No it's standardized. Outside of accidents the power providers take extreme care to keep the frequency synchronized to UTC. Exactly.

A clock using the power grid as reference might be off by 4 minutes over the day but on average it will be perfectly on time.


It used to be in the US that some operators would have a plain off-the-shelf alarm clock in the control room, and, in case the evening load made the frequency dip, operators would adjust things so that the clock would read 6am (for example) exactly the next day.


I think it is really old and cheap clocks (such as old radio alarm clocks) which use AC frequency for the clock signal. Their power consumption on AC is also rediculous compared to what they do (displaying the time).


I think the only clock I have that does it that way is the one in the stove/oven as I recently noticed the deviation after having set it correctly just a few weeks ago.


The real challenge is to have all clocks within a building display the same time. Getting them powered and synchronized through the electric grid is a straightforward solution.


No but things like heating systems are presumably designed for 20 year + lifespans, so there is probably a large number of legacy systems out there and will be for some time.


AC frequency makes the entire process cheaper.

You can use a resistor to lower the voltage and plug in a stepper motor and there is your clock.

When the idea was initially proposed quartz generators were also a bit more expensive so using the grid to synchronize was not a bad idea at all.

Even today it's not the worst idea since it's being kept very close to 50hz. Close enough that you can safely operate an alarm clock from it without being off to far by the end of the day.


You wouldn't use a resistor to lower the voltage, because then you have most of the power being dissipated in the resistor rather than the stepper motor. Much easier to just use a transformer or a 240V-rated stepper motor.


Well, normally you wouldn't draw power over that resistor, I was thinking more in the direction of using the power line as a "sense" pin for the stepper and power was provided properly via a transformer.


Before being relegated to cheapest chinese stuff, elecromechanical timers were the norm.

There's probably thousands of electrical panels with old DIN rail timers that don't get replaced until they die.


I was thinking the exact same thing. I've always assumed that AC frequency wasn't exactly 50Hz and was allowed to fluctuate slightly.


That assumption is right. But until now it was always compensated, so that in the mean it would be ~50hz. This correction mechanism apparently isn't enough anymore for the current fluctuations in europe.


Not mean, average. That's a very important distinction because if it was mean, then it might never correct, but it does.


It's supposed to average 50Hz over time, so using it for clocks make perfect sense.


It can be allowed to fluctuate, but still regulated to keep pretty much exactly 50Hz mean value over time.


Related (and 7 years old) article concerning US grid: "Power grid change may disrupt clocks" [0]. It was on HN [1], but with just two comments.

[0] https://web.archive.org/web/20110701065839/http://old.news.y...

[1] https://news.ycombinator.com/item?id=2703264


Discussion elsewhere has turned up when this happened in the UK https://books.google.co.uk/books?id=xnxQfGl8qfkC&pg=PA3&lpg=...


International disagreements in power supply, loop flows on HVDC lines, incentives for distributed generation that trigger huge distribution build outs that bankrupt utilities, nuclear shutdowns that make it hard to meet carbon goals—its an interesting time on the European grid.

Not that the North American grid doesn’t have problems. But the natural gas boom there has provided a nifty way to meet regulatory goals (carbon emmissions) with generator technology that’s familiar and very easy to build/control.


how can energy go missing because of an energy war? I found this but I don't understand: "Kosovo has been using more power than it produces while Serbia, responsible for balancing the Kosovo grid, has failed to do this, Nies said. As a result, Serbia has been free-riding on the system."


Suppose you and I have an agreement to generate power for our neighboring households. We each have solar which works most the time, but if it's too cloudy and we're both using a little more, we'll each fire up some temporary generators to cover the loss.

You think I'm using more power than you expected when we started the agreement, but we're both spending the same amount on fuel and solar, so you decide not to fire up your generator.

The system might still have enough power to basically run everything, but it might be sluggish, you might get "brown outs" which is when lights are on but dimmer, rather than going all the way to black.

A brown out is a drastic, noticeable reduction in frequency. The grid is a bit like a wind up toy where there's a lot of potential energy in the system, but as you extract that energy for work and don't replace it, things (machines, lights) slow down before they stop.

Now scale this up from two households to millions. Deviations in frequency might be incredibly slight, and not noticeable to the naked eye. But they accumulate over time if you've tied precision equipment to the grid.

Theoretically, certain industrial machines designed to run at specific frequencies will wear faster or break when run off frequency for too long. I'm not sure if those effects generally happen at such small deviations or not.


Basically, there are people using power on the grid without paying anyone to generate a corresponding quantity.

This means that less power is going into the grid than is being taken out of it, an amount that has accumulated since January to a bit over 100GWh. This missing 100GWh of energy has been collectively sapped from the inertia of the all spinning synchronous machines connected to the grid - mostly generators, but also motors - as a result of which they are spinning slower than they should be. Slowing spinning synchronous generators means a lower grid frequency.


Thanks, so its basically stolen at the generation level and thus you can't bill them 100GWh because that energy has never even appeared on the grid and technically nobody has consumed it.


No, it has definitely been consumed.

The only impediment to billing them is political.


Here's a great movie illustrating how the grid is balanced when a popular tv show ends and millions of people want a cup of hot tea: https://www.youtube.com/watch?v=slDAvewWfrA


I'm usually against technical solutions for political problems, but in this case it seems like a no-brainer to switch to cheap and ubiquitous crystal oscillators to decouple timekeeping accuracy from regional political stability.


AC grid frequency is much more stable than any crystal oscillator. After any number of years, AC will, on average, drift 0 seconds from UTC. It's being sync'd using atomic clocks to keep it as perfect as possible precisely because people use it as clock.


Which is better depends on whether you need short term accuracy or long term. On the long term, grid clock is more accurate. In the short term, crystals are more accurate.


This. In Florida US grid zero crossings randomly bounce around +/- 20nS over a few seconds. Viewing the waveform on a scope can be enlightening (or inflammatory and shocking if one's not properly isolated)


I would be willing to bet that this is noise.


The benefits to keeping timekeeping tied to power is that often the power supply is much more accurate than any reasonably priced quartz oscillator you'd want to put into an appliance.


I don't think the timekeeping is the part to be concerned about.


I sort of wonder how much of a problem that is any more. I have exactly zero clocks that aren't NTP enabled, radio synchronized or just dumb battery watches.

Still an interesting "problem" non the less.


Those clocks being off is just a slight annoyance, the actual problem is the threat to grid stability.


Good example of how a critical external dependency without an SLA is a problem waiting to happen.


Well, the way I read the article, is that one area is violating the SLA, albeit informally specified.


As long as it's not 2^32 seconds (~136 years) late/early, NTP will still be able to save the devices it runs on! [0]

[0] https://rachelbythebay.com/w/2017/09/27/2153/


These are devices from simpler times: no networking stack, not even an OS, sometimes not even firmware, just the basic function encoded with transistors. There's literally nothing to run on. Think water heaters, not RPi.


I know! Though strangely, my own kitchen appliances didn't participate in the continental time-shift (my microwave oven is still ticking correctly).

And my remark was more: even NTP can't save ou if you shift the time too much, so it's a good thing it's only ~6 minutes so far.


> even NTP can't save ou if you shift the time too much, so it's a good thing it's only ~6 minutes so far.

Only for the initial step, or if the frequency of the quartz crystal is too fast/slow. The NTP daemon continuously adjusts the clock frequency, so if it was already running and synchronized, the shift will never be more than a few milliseconds.


Probably not considering some of these used a stepper motor to drive their clock that was coupled to the grid frequency.

These devices don't have any computing power.


NTP, but for electrical grids


connected botnet ready clocks, but for electrical grids?

Or perhaps you just meant setting up a (non-internet) grid time protocol? It would be a risky venture, I'm guessing, as there may be other functions that rely on the 50Hz grid consistency that would also then need their own special protocol on the grid.


Well, there's already the peak/off-peak signalling. You could theoretically send another signal, perhaps with time encoded, at midnight or some such. Note that this is a unidirectional broadcast, as opposed to the complexity of an actual bidirectional network.


This has been annoying me since it started! The drift is nuts, I have to readjust the clock of my microwave because it's so much delayed and I always think it's earlier than it actually is!


Time is relative, you know? :) Maybe time in Bulgaria (where I live) indeed goes slower. p.s. I'll probably get downvotes for my comment..


The headline is kind of clickbait, almost no clocks in daily life get their base clock frequency from powerlines.

Edit: Nevermind


In Holland on the first day of the month we set off all the war alarms for a minute to test if they still function. This month a lot of them failed, caused by the power frequency


Imagine for a second that North Korea or US were involved in the electricity dispute in question... And because of that, as you say, Holland wan't test it's war alarm (which is vital infrastructure I guess) I wonder what would be the position of EU...


Do you really think the US is not involved in this dispute?


ahahaha conspiration !!!


it's not really a conspiracy when energy payment squabbles are smacking you in the face in the news..

https://www.export.gov/article?id=Kosovo-Energy

https://www.reuters.com/article/us-ukraine-crisis-russia-gaz...

etc etc etc


Oh god, I hope no Germans are reading this, it would be their perfect time to strike...


Where this procedure is in effect, there's a different signal for "test" and "emergency". The opposite situation is more common - we've had a couple of cases where the person in charge actually initiated the emergency protocol instead of the expected test protocol.


Not true. I experience this right now with the integrated clock in my stove. Applies to a lot of clocks in kitchen appliances.


What I read in this thread, heaters and whatnot too - at least kitchen clocks are visible and people would just reset them.


Also my digital clock radio is affected, I didn't know it was using AC like that until now


You might want to look around a bit, your assumption is incorrect: there's a surprising amount of appliances that are not IoT, but do rely on such unexpected side channels such as AC-as-a-clock - not everything has a network port, a quartz oscillator or even a GPS receiver, but most appliances do have a power plug.


It might interest you that you can use the power plug to connect the device to the internet.


Again, you are assuming that everything has enough power to run a networking stack, or even that there is Turing-equivalent hardware to run anything at all. (Yes, yes, I know this is HN, where any device is fit for a computer; most of the world isn't.) My water heater from late 1990s has a cleverly wired set of transistors - not even chips, the logic is literally wired into the product: cheap yet resilient. Connecting that to the internet would need adding some sort of computing device, adding several orders of magnitude in complexity (you can ping it and now it's also brittle, yay).

(I assume that you mean "to the local network, which is then connected via more conventional means, oh, and did I mention you need yet another computer to route data to the remote end?" - from what I've seen, attempts at WAN-over-power have largely been abandoned, even industrial IoT went for LoRa and friends. And from what I've experienced, LAN-over-power is a last-resort desperate hack: better than nothing at all, but barely better than two cans on a string w/r/t packet loss.)


I'm not assuming anything, I'm not participating in this discussion (and I agree with you). I'm just sharing an interesting information for the GP because it's not well known. You're right that it's not really usable in production; but it's nice for some simple home automation.


Ok, thanks for the correction. I thought I had overlooked some new development in the WAN-over-power :)


That is definitely interesting, do you know where I could find more info on it?


Sure, it's called "Powerline networking", you can find cheap modems that you connect to your internet connection and then you can use other modems as clients.


yeah except the one in my oven, in my microwave and in my coffee machine... all late by exactly the same amount.


Basically every device that is connected to main power and has a clock skips an internal quartz based circuit. Usually, this gives much higher precision for the same price point because frequency is typically controlled very, very tightly.


same for me, drove me nuts. Fortunately my PC looks at atomic clocks...


Your PC also has a quartz oscillator and battery backing, there's no way it synchronizes time from AC power.


sure, I just meant my PC has a very sophisticated (hence fragile) way of keeping its time accurate.

And forget about battery backing, it's a desktop and its bios battery is long dead :-) (hence the atomic clock)


Once you do synchronize it off the net, it's still the quartz oscillator keeping the local beat, as it were :)


I synchronize with ntp every 500 million cpu cycles :p


IIRC modern operatings systems use counters in the CPU for timekeeping instead of directly accessing the RTC


Well, the CPU is somewhat beyond my ken. You are right, of course - yet I think there would be another oscillator driving the clock generator as well. Turtles all the way down ;)


The clocks thing isn't quite true but you're right that it's a badly editorialized clickbait title. The linked press release never mentions anything as dramatic as 'an energy war in the Balkans' nor does the actual title reflect what's been posted on HN at all.


The clock thing is true, albeit simplified: I didn't have enough characters to say which type of clocks.

Regarding "an energy war in the Balkans", I could have said "an electricity dispute between Serbia and Kosovo", but again, I wouldn't have had enough characters.

I'm not sure the title was that bad, or that editorialized.


It was bad and editorialized, which is why the mods changed it. You could have just summarized the original title, as the site guidelines ask you to.


This is untrue given the examples in the article such as oven clocks




Guidelines | FAQ | Lists | API | Security | Legal | Apply to YC | Contact

Search: